LSAT and Law School Admissions Forum

Get expert LSAT preparation and law school admissions advice from PowerScore Test Preparation.

User avatar
 Dave Killoran
PowerScore Staff
  • PowerScore Staff
  • Posts: 5849
  • Joined: Mar 25, 2011
|
#94174
Complete Question Explanation
(The complete setup for this game can be found here: viewtopic.php?f=385&p=94172)

The correct answer choice is (A)

As established during our discussion of the numbers in this game, there are at most three technicians. Thus, answer choice (A) is correct.
 Paul Marsh
PowerScore Staff
  • PowerScore Staff
  • Posts: 290
  • Joined: Oct 15, 2019
|
#77867
This is a Global Must Be True question. A quick glance at our answer choices reveals that each choice is discussing the possible numeric distributions of our game. From our inferences during the Setup portion, we know that 1-2-2 and 1-1-3 are the only possible distributions. I scan the answer choices for the one that comports with those possible distributions. Since there can only be either 2 or 3 technicians, (A) must be our correct answer.

Get the most out of your LSAT Prep Plus subscription.

Analyze and track your performance with our Testing and Analytics Package.